2
$\begingroup$

Let $D\subseteq X$ be a dense subset of a separable metric space $X$. Let $P(D)$ and $P(X)$ respectively denote the probability measures on $D$ and on $X$ with their weak topologies. Then, if we view $P(D)$ as a subset of $P(X)$ via the "inclusion" $\iota:P(D)\rightarrow P(X)$ defined by: $$ \iota(\mu)\mapsto \mu(\cdot \cap D) $$ is this subset dense?

$\endgroup$
2
  • $\begingroup$ You would have to specify which topology you are using--there are two natural ones, for one it is true, for the other false. $\endgroup$ Feb 19, 2021 at 11:26
  • $\begingroup$ The weak topology (obviously it is false in TV) $\endgroup$
    – ABIM
    Feb 19, 2021 at 11:35

1 Answer 1

3
$\begingroup$

Yes. First the probability measures with finite support are dense in $P(X)$. Second, if $P = \sum_{i=1}^n p_i \delta_{x_i}$ with $x_i \in X$ and $\sum_{i=1}^n p_i = 1$, let $(x_{im})_{m \in \mathbb{N}}$ be sequences in $D$ with $\lim_{m \to \infty} x_{im} = x_i$. Then $\lim_{m \to \infty} \sum_{i=1}^n p_i \delta_{x_{im}} = P$.

$\endgroup$

Your Answer

By clicking “Post Your Answer”, you agree to our terms of service and acknowledge that you have read and understand our privacy policy and code of conduct.

Not the answer you're looking for? Browse other questions tagged or ask your own question.